Two Conjectures about Permutations in ##S_n##

In summary, the first conjecture states that if ##\tau, \sigma \in S_n## are disjoint, then ##\tau \sigma = e## if and only if ##\tau = e## and ##\sigma = e##. The second conjecture states that if ##\tau## and ##\sigma## are disjoint, then ##\tau^n## and ##\sigma^n## are disjoint for every integer ##n##.
  • #1
Bashyboy
1,421
5

Homework Statement


I have two conjectures whose truth I am interested in. The first is, "If ##\tau, \sigma \in S_n## are disjoint, then ##\tau \sigma = e## if and only if ##\tau = e## and ##\sigma = e##., where ##e## denotes the identity permutation." The second is, "If ##\tau## and ##\sigma## are disjoint, then ##\tau^n## and ##\sigma^n## are disjoint for every integer ##n##.

Homework Equations

The Attempt at a Solution



I think I may have a proof of the first, but I am hoping someone can help me clean it up and clarify a few notions: Suppose that ##\tau## and ##\sigma## are disjoint, satisfying ##\tau \sigma = e##. Then ##(\tau \circ \sigma)(a) = \tau(\sigma(a)) = a## for all ##a \in \{1,...,n\}##. Let ##a## be a positive integer not appearing in ##\sigma##'s cycle decomposition but in ##\tau##'s. Then ##\tau(\sigma(a)) = a## or ##\tau(a) = a##, which is a contradiction unless ##\tau = e## and therefore ##\sigma = e##.

What exactly does it mean for "##a## not to appear on the cycle decomposition?" I can construe that as meaning ##\tau(a) = a##.

Here is an attempt at proving the second conjecture: Suppose that ##\tau## and ##\sigma## are disjoint permutations. When ##n=1##, the claim follows trivially. So suppose the claims holds for all such cycles raised to the ##n## power. Then ##\tau^{n+1} = \tau^n \tau## and ##\sigma^{n+1} = \sigma^n \sigma##. By the induction hypothesis, ##\tau_n## and ##\sigma^n## are disjoint...

At this point, it seems that I would need something like the following to be true: If ##\tau_1##, ##\sigma_1## and ##\tau_2##, ##\sigma_2## are pairs of disjoint permutations, then ##\tau_1 \tau_2## and ##\sigma_1 \sigma_2## are disjoint permutations. Before I go any further, however, I would like to have my one proof verified and all these conjectures corroborated.

Thanks!
 
Physics news on Phys.org
  • #2
Bashyboy said:
Suppose that ##\tau## and ##\sigma## are disjoint, satisfying ##\tau \sigma = e##. Then ##(\tau \circ \sigma)(a) = \tau(\sigma(a)) = a## for all ##a \in \{1,...,n\}##. Let ##a## be a positive integer not appearing in ##\sigma##'s cycle decomposition but in ##\tau##'s. Then ##\tau(\sigma(a)) = a## or ##\tau(a) = a##, which is a contradiction unless ##\tau = e## and therefore ##\sigma = e##.
Looks good, except one point: Why there has to be an element ##a##?
(I know why, but you should mention this (im-)possibility.)
Bashyboy said:
If ##\tau_1##, ##\sigma_1## and ##\tau_2##, ##\sigma_2## are pairs of disjoint permutations, then ##\tau_1 \tau_2## and ##\sigma_1 \sigma_2## are disjoint permutations.
This isn't true, because ##\tau_2=\sigma_1## or ##\tau_1=\sigma_2## could be the case and there is no assumption on them.
Why don't you perform a double induction, first on ##\tau^n## and then on ##\sigma^n##? Just an idea, because the statement is also true for different powers. Or operate with the cycle decomposition and commutativity of ##\tau## and ##\sigma##.
 
  • #3
fresh_42 said:
Looks good, except one point: Why there has to be an element aaa?
(I know why, but you should mention this (im-)possibility.)

I am not sure how to justify this. I suppose I would say that, since ##\tau \neq e## and ##\sigma \neq e## (otherwise the claim follows trivially), then at least two integers appear in the cycle decomposition of each permutation. Moreover, since ##\tau## and ##\sigma## are disjoint, then every integer in ##\tau##'s cycle decomposition does not appear in ##\sigma##'s cycle decomposition. It seems that these two facts coupled together guarantee the existence of at least integer in ##\tau##'s decomposition that is not in ##\sigma##'s.

Before I offer a proof of the second claim, let me first prove a lemma: If ##\tau## and ##\sigma## are disjoint, then their inverses ##\tau^{-1}## and ##\sigma^{-1}## are disjoint. Assume the hypothesis, but suppose that the inverses are not disjoint. Then there exists an ##a## in both decompositions such that ##\tau^{-1}(a) = b## and ##\sigma^{-1}(a) = c##, where ##b## and ##c## are not ##a##. Note the following two equations hold if and only if ##a = \tau(b)## and ##a = \sigma(c)##. Since ##\tau## and ##\sigma## are bijections, neither can map ##a## to ##a##, implying that ##a## appears in the cycle decomposition of both ##\tau## and ##\sigma##, contradicting initial assumption. Hence, the inverses must be disjoint.

Now, here is a proof of the second conjecture: We prove that if ##\tau## and ##\sigma## are disjoint, then ##\tau^m## and ##\sigma^n## are disjoint for all integers ##m## and ##n##.

First we establish the result for natural numbers through induction. Clearly when ##m=n=1##, the claim holds trivially. Now, suppose the claim holds for ##m=1## and some arbitrary ##n \in \mathbb{N}##, but suppose that ##\tau## and ##\sigma^{n+1}## not disjoint. Then there exists a natural number ##a## such that ##\tau(a) \neq a## and ##\sigma^{n+1}(a) \neq a## or ##\sigma(\sigma^n(a) \neq a##. By hypothesis, ##\sigma^n## and ##\tau## are disjoint, implying ##\sigma^n(a) = a##. Hence, ##\sigma(a) \neq a##, implying that ##a## appears in ##\sigma##'s cycle decomposition, a contradiction.

Hence, we have that if ##\tau## and ##\sigma## are disjoint, then ##\tau## and ##\sigma^n## are disjoint for all ##n \in \mathbb{N}##. By symmetry, we also have the conclusion that ##\tau^m## and ##\sigma## are disjoint for all ##m \in \mathbb{N}##. Hence, ##\tau^m## and ##\sigma^n## are disjoint for all (strictly speaking, one needs that disjointness is transitive, but I'll just omit that; I have already proven enough). Now we prove it for negative integers.

Suppose that ##m,n \in \mathbb{Z}^-##. Then ##-m,-n \in \mathbb{N}##, and therefore ##\tau^{-m}## and ##\sigma^{-n}## are disjoint. Finally, by the above lemma, their inverses ##\tau^m## and ##\sigma^n## are disjoint.

Whew! That took quite a long time to type, but it was certainly worthwhile. So, how do these proofs sound?
 
  • #4
Bashyboy said:
I am not sure how to justify this. I suppose I would say that, since ##\tau \neq e## and ##\sigma \neq e## (otherwise the claim follows trivially), then at least two integers appear in the cycle decomposition of each permutation. Moreover, since ##\tau## and ##\sigma## are disjoint, then every integer in ##\tau##'s cycle decomposition does not appear in ##\sigma##'s cycle decomposition. It seems that these two facts coupled together guarantee the existence of at least integer in ##\tau##'s decomposition that is not in ##\sigma##'s.
I agree.
Before I offer a proof of the second claim, let me first prove a lemma: If ##\tau## and ##\sigma## are disjoint, then their inverses ##\tau^{-1}## and ##\sigma^{-1}## are disjoint. Assume the hypothesis, but suppose that the inverses are not disjoint. Then there exists an ##a## in both decompositions such that ##\tau^{-1}(a) = b## and ##\sigma^{-1}(a) = c##, where ##b## and ##c## are not ##a##. Note the following two equations hold if and only if ##a = \tau(b)## and ##a = \sigma(c)##. Since ##\tau## and ##\sigma## are bijections, ...
##(12) \in S_3## is also a bijection, but maps ##3## onto itself, so the argument doesn't apply. However, e.g. ##a \notin \tau \Longleftrightarrow \tau(a)=a \Longleftrightarrow \tau^{-1}(a)=a \Longleftrightarrow a \notin \tau^{-1}## because the cycle decomposition is a decomposition in disjoint cycles so ##a## can appear in at most one cycle and thus cannot be invariant.
... neither can map ##a## to ##a##, implying that ##a## appears in the cycle decomposition of both ##\tau## and ##\sigma##, contradicting initial assumption. Hence, the inverses must be disjoint.

Now, here is a proof of the second conjecture: We prove that if ##\tau## and ##\sigma## are disjoint, then ##\tau^m## and ##\sigma^n## are disjoint for all integers ##m## and ##n##.

First we establish the result for natural numbers through induction. Clearly when ##m=n=1##, the claim holds trivially. Now, suppose the claim holds for ##m=1## and some arbitrary ##n \in \mathbb{N}##, but suppose that ##\tau## and ##\sigma^{n+1}## not disjoint. Then there exists a natural number ##a## such that ##\tau(a) \neq a## and ##\sigma^{n+1}(a) \neq a## or ##\sigma(\sigma^n(a) \neq a##. By hypothesis, ##\sigma^n## and ##\tau## are disjoint, implying ##\sigma^n(a) = a##. Hence, ##\sigma(a) \neq a##, implying that ##a## appears in ##\sigma##'s cycle decomposition, a contradiction.
The fact, that ##\tau## and ##\sigma## are disjoint can be written as ##\forall b \, : \,\tau(b) \neq b \Rightarrow \sigma(b) = b##.
(I only mentioned this as a remainder for me, because I can better grasp it in this notation.)
Hence, we have that if ##\tau## and ##\sigma## are disjoint, then ##\tau## and ##\sigma^n## are disjoint for all ##n \in \mathbb{N}##. By symmetry, we also have the conclusion that ##\tau^m## and ##\sigma## are disjoint for all ##m \in \mathbb{N}##. Hence, ##\tau^m## and ##\sigma^n## are disjoint for all (strictly speaking, one needs that disjointness is transitive, but I'll just omit that; I have already proven enough). Now we prove it for negative integers.

Suppose that ##m,n \in \mathbb{Z}^-##. Then ##-m,-n \in \mathbb{N}##, and therefore ##\tau^{-m}## and ##\sigma^{-n}## are disjoint. Finally, by the above lemma, their inverses ##\tau^m## and ##\sigma^n## are disjoint.

Whew! That took quite a long time to type, but it was certainly worthwhile. So, how do these proofs sound?
Under the assumption I didn't get lost in the middle, yes. But strictly speaking: disjointness isn't transitive:
##((12),(34)) = 1 \wedge ((34),(15))=1 \wedge ((12),(15)) \neq 1##.

I'm sure it could be seen in a shorter way, e.g. by the equivalences I wrote above. But at least it was a good exercise - although I feel a bit dizzy now.
 

Related to Two Conjectures about Permutations in ##S_n##

1. What are the two conjectures about permutations in ##S_n##?

The two conjectures are the Stanley-Wilf Conjecture and the Simion-Ulman Conjecture. The Stanley-Wilf Conjecture states that for any two permutations ##π## and ##σ## in ##S_n##, the number of occurrences of ##π## in a random permutation of length ##n## is asymptotically equal to the number of occurrences of ##σ##. The Simion-Ulman Conjecture states that for any two permutations ##π## and ##σ## in ##S_n##, the number of occurrences of ##π## in a random permutation of length ##n## is maximized when ##σ## is the identity permutation.

2. What is the significance of these conjectures?

These conjectures have important implications in various fields, including combinatorics, computer science, and statistical mechanics. They provide insight into the structure and behavior of permutations, and have been used to solve problems in scheduling, network routing, and DNA sequencing.

3. What evidence exists for these conjectures?

Several studies have been conducted to test these conjectures, and the evidence strongly supports their validity. In particular, the Stanley-Wilf Conjecture has been proven for certain classes of permutations, such as pattern-avoiding and monotone permutations. The Simion-Ulman Conjecture has also been proven for various classes of permutations, including cyclic and layered permutations.

4. Are there any known counterexamples to these conjectures?

No counterexamples have been found for either conjecture. However, it is important to note that the conjectures have only been proven for certain classes of permutations, and it is possible that there may exist counterexamples for other types of permutations.

5. What are the potential applications of these conjectures?

The applications of these conjectures are numerous and varied. They have been used to solve problems in computer science, such as designing efficient algorithms for sorting and searching. They have also been applied in the study of random graphs and networks, as well as in statistical mechanics to model the behavior of physical systems.

Similar threads

  • Linear and Abstract Algebra
Replies
7
Views
640
  • Calculus and Beyond Homework Help
Replies
7
Views
1K
  • Calculus and Beyond Homework Help
Replies
4
Views
1K
  • Calculus and Beyond Homework Help
Replies
11
Views
1K
  • Calculus and Beyond Homework Help
Replies
3
Views
788
  • Topology and Analysis
Replies
2
Views
2K
  • Calculus and Beyond Homework Help
Replies
4
Views
2K
  • Calculus and Beyond Homework Help
Replies
4
Views
854
  • Calculus and Beyond Homework Help
Replies
2
Views
1K
  • Calculus and Beyond Homework Help
Replies
2
Views
1K
Back
Top